Publications MATHS-LYCEE.FR

mémo+exercices corrigés+liens vidéos

L'essentiel pour réussir la première en spécialité maths

RÉUSSIR EN MATHS, C'EST POSSIBLE!
Tous les chapitres avec pour chaque notion:
- mémo cours
- exercices corrigés d'application directe
- liens vidéos d'explications.
Il est indispensable de maîtriser parfaitement les notions de base et leur application directe pour pourvoir ensuite les utiliser dans la résolution de problèmes plus complexes.

Plus d'infos

Aide en ligne avec WhatsApp*, un professeur est à vos côtés à tout moment! Essayez!
Un cours particulier à la demande!

Envoyez un message WhatsApp au 07 67 45 85 81 en précisant votre nom d'utilisateur.
*période d'essai ou abonnés premium(aide illimitée, accès aux PDF et suppression de la pub)
PDF reservé aux abonnés
Un entreprise fabrique des climatiseurs et le coût total de production mensuel, en euros, en fonction du nombre $q$ de climatiseurs fabriqués est donné par $C(q)=q^3-300q^2+25000q$.
L'entreprise a une capacité de production mensuelle maximale de 300 climatiseurs.
Chaque climatiseur est vendu 8900 euros et on suppose que toute la production est vendue.
  1. Exprimer en fonction de $q$ le coût de production $C_m(q)$ d'un climatiseur (coût moyen de production) quand on en produit une quantité $q$ dans le mois.
    Le coût moyen par climatiseur est le coût total divisé par le nombre total de climatiseurs produits.
    $C_m(q)=\dfrac{C(q)}{q}=\dfrac{q^3-300q^2+25000q}{q}=\dfrac{q(q^2-300q+25000)}{q}=q^2-300q+25000$
  2. Vérifier que $C_m(q)=(q-150)^2+2500$.

    Identités remarquables


    $(a+b)^2=a^2+2ab+b^2$
    $(a-b)^2=a^2-2ab+b^2$
    $(a-b)(a+b)=a^2-b^2$
    On peut développer l'expression proposée
    $(q-150)^2+2500=q^2-2\times q\times 150+150^2+2500=q^2-300q+25000$
  3. Calculer $C_m(150)$.
    et en déduire que le minimum de la fonction $C_m$ est 2500 pour une production mensuelle de 150 climatiseurs.
    Quel est alors le bénéfice mensuel de l'entreprise?

    Comparer deux nombres


    Soit $a$ et $b$ deux nombres réels, $a < b$ si et seulement si $b-a>0$
    Conséquence: Pour comparer deux nombres ou deux expressions, on peut étudier le signe de leur différence.
    On veut comparer $C_m(q)$ et $2500$ en utilisant l'expression de la question précédente.
    $C_m(150)=(150-150)^2+2500=0+2500=2500$
    $C_m(q)-2500=(q-150)^2+2500-2500=(q-150)^2$
    Un carré est toujours positif donc $C_m(q)-2500\geq 0$
    donc $C_m(q)\geq 2500$ et on a de plus $C_m(150)=2500$

    La recette totale est alors de $8900\times 150=1~335~000$ euros
    pour un coût total de fabrication de $C(150)=150^3-300\times 150^2+25000\times 150=375~000$ euros.
    La recette est donc de $1~335~000-375~000=960~000$


    Cela signifie que le coût minimum de production par climatiseur est de 2500 euros et cela lorsque l'on en produit 150.
  4. Exprimer, en fonction de $q$, la recette totale mensuelle de l'entreprise.
    La recette correspond à la somme récoltée pour $q$ climatiseurs vendus.
    Chaque climatiseurs est vendu 8900 euros
  5. Exprimer le bénéfice total mensuel $B(q)$ de l'entreprise en fonction de $q$.
    Le bénéfice est égal à la recette diminuée des coût de production
    $B(q)=R(q)-C(q)=8900q-(q^3-300q^2+25000q)$
    $\phantom{B(q)}=8900q-q^3+300q^2-25000q$
    $\phantom{B(q)}=-q^3+300q^2-16100q$
  6. Exprimer le bénéfice de l'entreprise $B_m(q)$ sur chaque climatiseur(bénéfice moyen) en fonction de $q$ quand la production mensuelle est de $q$ climatiseurs.
    Le bénéfice moyen correspond au bénéfice total divisé par le nombre total de climatiseurs vendus.
    $B_m(q)=\dfrac{B(q)}{q}=\dfrac{-q^3+300q^2-16100q}{q}=\dfrac{q(-q^2+300q-16100)}{q}=-q^2+300q-16100$
  7. Montrer que pour tout réel $q\in[0;300]$, $B_m(q)=6400-(q-150)^2$ puis factoriser $B_m(q)$
    On peut développer l'expression proposée
    signe $-$ devant la parenthèse
    $6400-(q-150)^2=6400-(q^2-300q+150^2)=6400-q^2+300q-22500=-q^2+300q-16100$
  8. Résoudre $B_m(q)\geq 0$ et en déduire le nombre d'objets à fabriquer pour que l'entreprise ne soit pas en déficit.

    Identités remarquables


    $(a+b)^2=a^2+2ab+b^2$
    $(a-b)^2=a^2-2ab+b^2$
    $(a-b)(a+b)=a^2-b^2$
    Il faut factoriser $B_m(q)$ en utilisant la question précédente
    $B_m(q)=6400-(q-150)^2$
    $\phantom{B_m(q)}=80^2-(q-150)^2$
    $\phantom{B_m(q)}=(80-q+150)(80+q-150)$
    $\phantom{B_m(q)}=(230-q)(q-70)$
    $230-q$ s'annule pour $q=230$ et $q-70$ s'annule pour $q=70$

    donc $B(q)\geq 0$ (zone bleue) pour $q\in [70;230]$
    L'entreprise n'est pas en déficit si $B(q)\geq 0$
  9. On donne ci-dessous les représentations graphiques $C_C$ et $C_R$ respectivement des fonctions coût ($C$) et recette ($R$).

    Comment peut-on contrôler le résultat de la question précédente?
    L'entreprise n'est pas en déficit si la recette est supérieure au coût de production.
    L'entreprise n'est pas en déficit si la recette est supérieure au coût de production
    donc il faut résoudre $R(q) \geq C(q)$.
    Graphiquement, les solutions de l'inéquation $R(q) \geq C(q)$ sont les abscisses (en vert) des points de la courbe $C_R$(en bleu) situés au-dessus de la courbe $C_C$


Attention les fonctions ci-dessus sont désactivées en mode "visiteur", créez un compte MATHS-LYCEE.FR (gratuit)